QUESTION TEXT: If Skiff's book is published this year, Professor…
QUESTION TYPE: Sufficient Assumption
CONCLUSION: Skiff will be promoted if his book is as important and as well written as he claims.
REASONING: Published this year ➞ Recommendation ➞ Promotion
ANALYSIS: This is a trap question. It’s designed to catch people who have an incorrect, rigid view of logic.
We’re trying to reach “promotion”. And the conclusion is “important AND well written ➞ promotion”. You might think that you need both important and well written to join the conclusion to the evidence. But that’s not the case. We’re just trying to show that if the book has both those things, then Skiff will be promoted.
Answer A is correct because it makes the evidence chain look like this: Important ➞ published this year ➞ Recommendation ➞ Promotion
So if the book is important, Skiff will be promoted. In this chain, we only need “important”.
Lets look at the conclusion again. It says that if the book is important and well written, skiff is promoted. If the book is important and well written, is the book important? Yes. And that’s now the sufficient condition we need.
Three of the wrong answers add necessary conditions. Those never help you prove something will happen. They can only prove something won’t happen, when the necessary condition is missing.
___________
- CORRECT. See the explanation above.
- This doesn’t tell us Skiff will publish in time to get Nguyen’s recommendation. It just adds another useless necessary condition Skiff needs to fulfill before getting promoted.
- It doesn’t matter whether Nguyen thinks the book is well written. We already know Nguyen will recommend Skiff once he publishes.
- Same as B. This adds an additional necessary condition. That doesn’t help us prove that the book will be published this year.
- Same as B and D. This adds a necessary condition. That makes it harder for Skiff to get promoted.
Member Oscar Gonzalez says
I too was confused with the wording of “and” used here, but a way that helped me see it for what it is was to just separate the two terms.
The conclusion in the stimulus says that “IF Skiff’s book is as important AND as well written as Skiff claims, he will be promoted.”
Being that it’s a sufficient condition – hence the “IF” – you can actually diagram the two terms (important and well-written) separately because they’re both SUFFICIENT for being promoted:
1). important -> promoted
2). well-written -> promoted
Choice A) uses the first one – that’s all we need because, in EITHER one, you can still combine it with the premises and justify its conclusion.
I am NOT a tutor (in case I made some sort of mistake), just another student studying for the LSAT and figured I’d try to help.
Founder Graeme Blake says
This is precisely correct. Well put!
Note: This is an old comment but I wanted to clarify the point.
Member Claire Z. says
I’m passing along a tip that really helped me with sufficient assumptions.
I got this question right because, when I was learning about sufficient assumptions, I realized that “sufficient” means “enough.”
A sufficient assumption is an assumption that’s enough to get you to the conclusion.
Take this argument:
I feed my kids. I listen to their thoughts. I talk to aliens. Therefore, I am a good mother.
Here is a correct sufficient assumption answer choice:
People who talk to aliens are good mothers.
It doesn’t matter that my argument referred to more than one factor. If people who talk to aliens are good mothers, then boom, I’m there. That answer choice was enough to prove my argument.
Similarly, the argument may refer to two factors for the book (important and well-written), but as soon as you say the book WILL BE PUBLISHED IF IT IS IMPORTANT, boom, you’re there. It’s enough.
Tutor Lucas (LSAT Hacks) says
Hah, I like the example!
And this is a helpful write-up. Thanks Claire!
LT says
Had this been: Well Written —–> Skiff’s book Published, would that have been correct?
Tutor Lucas (LSAT Hacks) says
Yes. If (A) said, “Skiff’s book will be published if it is as well-written as Skiff claims it is”, then (A) would still be correct.
The argument would look like this:
Well-written –> Published –> Nguyen recommends –> Dean promotes Skiff
Jacob says
What I meant by my last question was, because we cannot fit the conclusion, as the author gave it to us, logically into our chain of logic, then we must consider important to be sufficient?
Because,
you cannot say:
recommendation -> Published -> Important -> promotion Because it breaks the chain. Published must go directly before recommendation.
Tutor Lucas (LSAT Hacks) says
Yes, we’re looking to create the following chain:
Book important –> Skiff published –> Promotion
(D) gives us Published –> Important
That’s a reversal of the missing link we’re looking for.
Jacob Menard says
But aren’t “important” and “well written” conditions for promotion?
How do they all of a sudden become conditions for publication?
Jacob Menard says
To me it doesn’t necessarily have to be important to be published…Couldn’t the book just be entertaining to the publisher? Not necessarily important? Unless because it’s an assumption question we’re supposed to make this assumption?
Tutor Lucas (LSAT Hacks) says
1) Here’s how “Important” and “Well-written” can become conditions for publication
This is a sufficient assumption question, so we’re just looking for any premise that, when added to those we’ve been given, will assure the truth of our conclusion.
We know from the stimulus that:
Skiff published –> Promotion
Because Nguyen will keep her promise to urge the dean to promote Skiff, and the dean will give the promotion if Nguyen recommends it.
Our task in this question is to prove that:
Book important + Well-written –> Promotion
So, the easiest thing to do is to somehow connect (using the answer choices)
Book important + Well-written –> Skiff published
Because then we get the following chain:
Book important + Well-written –> Skiff published –> Promotion
(A) provides the missing link: Book important –> Skiff published
Now we can create the following chain:
Book important –> Skiff published –> Promotion
This is exactly what our task in this question calls for. We don’t need the answer choice to also include “well-written” because as long as the book is important, it’s published.
2) You’re right, there are potentially a lot of other things that could make the book worth publishing. But on the LSAT, we’re asked to zero in on the stimulus as is. And this is a sufficient assumption question, so we’re looking for something that will help this particular argument as written to assure the certainty of its conclusion.
Jacob says
Okay so I looked at it again….
The question asks, the argument’s conclusion can be properly inferred if which one of the following is assumed?
So this means that we are looking to continue the chain of reasoning…We are looking for the assumption that will allow the conclusion to flow logically…
I think I get it now. So if you put answer choice D into this form of logic :
Not Important -> Not published and you CP it:
Publiched -> Important, you cannot plug this into the chain of reasoning. Therefore,
Important -> Published is the only which could be considered sufficient…Considering the conclusion that the author gave us, as well as the options?
So because we cannot fit the conclusion into the chain of logic, we must go with: important -> published?
Tutor Lucas (LSAT Hacks) says
Yes, that’s right re: sufficient assumption questions. You’re looking for an answer choice that when plugged into the argument in the stimulus will ensure that the truth of the premises assures the truth of the conclusion. And your analysis of why (D) is the incorrect answer is also correct.
Austin says
LSAT messed up on this one. To get published his book must be important AND well-written. It can be important and not well-written, and that does not satisfy Nguyen’s conditions.
Founder Graeme Blake says
I’m going to rewrite my explanation, since clearly my explanation was confusing. LSAC didn’t mess up. This question is a trap.
The gist is that we have this chain of evidence: published this year ➞ Recommendation ➞ Promotion
We want to get to promotion. The author is saying that if the book is important and well written, Skiff will be promoted.
Saying “if the book is important and well written, skiff will publish” would have been a correct answer. But you don’t need both parts. A says “important –> published.”. So we get this chain: important –> published this year ➞ Recommendation ➞ Promotion
The author’s conclusion is “if it’s important and well written, skiff will be promoted”. Well, now that’s true, because the chain above says “important”, on it’s own, is enough to lead to promotion. And if the book is important and well written, then it’s certainly important.
Joe says
Take question six in section four of this same test.
Selecting the correct answer requires one to make a distinction between “and” and “or”.
Member Sabrina (LSAT Hacks) says
In question 6, the main difference is the fact that the stimulus says “should not be ..unless.” This is essentially the same as saying “should be… if and only if.” This removes any ambiguity, and the difference between and/or matters a lot.
Joe says
Why is just one of them enought to prove the argument true?
Suppose I tell you three things:
1. The book is important.
2. The book is not well written.
3. If the book is important and well written, it will be published this year.
There is an important distinction between “and” and “or” that the LSAT often exploits on other questions, but I think they got it wrong on this particular question.
Member Sabrina (LSAT Hacks) says
Hi Joe,
Since the argument doesn’t say “if and only if the book is important and well written,” we can’t say that being both important and well written is a necessary condition for publishing. One of those things might be sufficient; the stimulus doesn’t lock down which one.
As Graeme explained the logical train of the correct answer choice goes something like this:
Important –> Published this year –> Recommend –> Promote
But it could just as easily have been:
Well written –> Published this year –> Recommend –> Promote
Or, as you said:
Both well written AND important –> Published this year –> Recommend –> Promote
Well written and important could each be sufficient conditions, independent of each other. If the book is both, Skiff will definitely be published, and therefore definitely promoted. If it’s one or the other, but not both, he’ll also be published, and also promoted. The conclusion could be properly inferred in all three situations (which is why they weren’t all answer choices).
I hope that helps!
Joe says
Hi Sabrina,
I appreciate your quick response. Maybe I am overthinking the question, but how can one be sure it is ok to assume that either condition is sufficient and not that both are necessary? Is it just based on outside knowledge and common sense?
For example one could replace “important” and “well written” with “finishes writing” and “submits for publication”.
Thanks for all the great explanations!
Member Sabrina (LSAT Hacks) says
Hey Joe,
It’s not so much outside knowledge as the way the question is worded.
We know that Skiff’s book getting published leads to a recommendation and a promotion. If the book has what it needs to get published, good things happen for Skiff
The wording does not tell us, however, that it is being BOTH important and well written that gets Skiff’s book published – just that IF we know it IS ‘important and well written,’ it has SOME quality that ensures it will be published. That quality could be any number of things (including important or well-written), and that’s what the question is asking for.
If the stim said Skiff’s book will be published if and only if it is important and well written, the word AND becomes more important. In that case, both are necessary.
Founder Graeme says
Hmm, my explanation isn’t as clear as it could be. I’m going to reword it – thanks for pointing this out.
You’re right to notice that the evidence says “important AND well written.”
However, just one of them is enough to prove the argument true. We already know this much:
“Published this year –> recommend –> dean will promote”
Then they say if “the book is important and well written”
Suppose I tell you three things:
1. The book is important
2. The book is well written
3. If the book is important, it will be published this year
We get:
Important –> Published this year –> Recommend –> Promote
Does that make sense? If either one of them is sufficient for the book to be published, then that’s all we need.
Rick says
‘However, just one of them is enough to prove the argument true. ‘
Agreed. But since we are to choose the ‘best’ answer, and answer D absolutely offers a more complete and correct answer, this question infuriates me. How can it possibly be argued that in an instance where the stimulus provides two necessary conditions in conjunction, and one of the answer choices calls back perfectly to both those conditions, that an answer choice which is virtually the same but calls back to only one of the two necessary conditions in the stimulus be considered the ‘best’ answer. Seriously, this question is infuriating. I was as sure that D was correct as I have ever been on any sufficient assumption question, and I simply cannot compute the notion that the much less complete answer of ‘A’ is actually correct.
All that aside, thanks so much for posting these explanations. They have been an incredible resource.
Founder Graeme Blake says
It isn’t more complete though. For example what if I said: “to get into law school you need to breathe and apply and eat food and sleep and own a car”
That doesn’t help you get into law school. Fulfilling always of those doesn’t get you in. But missing one disqualifies you.
We need a sufficient condition, for example “if you apply and you have a car you go to law school”. That means IF you apply and you have a car you do get in.
A is in the right format: it provides a sufficient condition, a qualifying condition. Whereas D provides two necessary conditions. They can only disqualify but never qualify you even if you meet them.
Note: This is an old comment but I wanted to clarify the point.
Tom B. says
But the stimulus doesn’t say important OR well written, it says important AND well written. (This question tripped me up on test day and I was thrown off by answer choice (A) because it didn’t include “and well written”).